Medical MCQs

Tech Books Yard have 1951 Medical for Free Download

Which of the following ECG leads is not unipolar ?

  • A. aVF
  • B. II
  • C. aVL
  • D. V3
  • Correct Answer: Option B

Which of the following is not a limb lead?

  • A. aVF
  • B. II
  • C. aVL
  • D. V3
  • Correct Answer: Option D

Which of the following ventricular myocardial segments is explored by the combinations of: II, III and aVF ECG leads ?

  • A. antero-lateral
  • B. antero-apical
  • C. inferior
  • D. antero-septal
  • Correct Answer: Option C

A 84 year old male with severe emphysema and a prior MI becomes short of breath with exertion. PE reveals a III/VI holosystolic murmur at the cardiac apex, a S3 heart sound, and rales in the lower lung fields. No lower extremity oedema is present. Which of the following is likely?

  • A. left-sided congestive HF
  • B. right-sided congestive HF
  • C. left and right sided congestive HF
  • D. Cor pulmonale
  • Correct Answer: Option A

A 35y old man presented with a 10-day history of progressive shortness of breath. He was free of chest pain, did not have any syncope. He was diagnosed with extra-hepatic portal vein thrombosis following episodes of haematemesis (vomiting blood) from 3 years ago, resulting in recurrent oesophageal varies requiring banding on several occasions. PE revealed blood pressure 100/60 mmHg, pulse rate of 88 beats per min, respiration rate of 16 breaths/min, JV distention, ascites, a narrow split second heart sound and accentuation of pulmonic closure on cardiac auscultation. TTE shows enlarged right chambers, moderate tricuspid RG with severe pulmonary hypertension. Although dilated there was no evidence of thrombus in the main right and left pulmonary arteries. Blood D-Dimer and fibrinogen were normal. Which examination would you recommend to reach diagnosis?

  • A. chest x-ray for pneumonia
  • B. pulmonary angiography for acute pulmonary embolism
  • C. CT pulmonary angiography for chronic thromboembolic pulmonary hypertension
  • D. right heart catheterization for pulmonary hypertension
  • Correct Answer: Option C

A 26 year old male is noted with a V/VI holosystolic murmur associated with a thrill at the left lower sternal border. He has no health complaints and is able to exercise regularly without difficulty. Which of the following is the likely diagnosis?

  • A. atrial septal defect
  • B. ventricular septal defect
  • C. mitralvalve RG
  • D. tricuspidvalve RG
  • Correct Answer: Option B

A 34y old male experiences shortness of breath with minimal exertion. PE, revels JVP markedly worse with inspiration, a regular rhythm with a S4 heart sound and 2 lower extremity pitting edema. Lab results are normal. Cardiac biopsy revealed apple green birefringence with congo red staining. Genetic testing reveals a mutation in the transthyretin gene. Which of the following is the correct diagnosis ?

  • A. restrictive cardiomyopathy
  • B. dilated cardiomyopathy
  • C. constrictive pericarditis
  • D. hypertrophic obstructive cardiomyopathy
  • Correct Answer: Option D

A 72y old female with a history of diabetes mellitus with no history of heart diseases goes to ER because of chest pain at rest for the past 4 hours. She has associated shortness of breath and diaphoresis. Her heart rate is 59, blood pressure 134/72, respiratory rate 20 and oxygen saturation 95%. On PE, normal lungs and S4 gallop. On ECG, ST segment depression in leads V1 to V3. She is given aspirin. Her troponin levels are negative. She is currently chest pain free. Which of the following is correct diagnosis?

  • A. silent ischemia
  • B. stable angina
  • C. unstable angina
  • D. non-ST segment elevation MI
  • Correct Answer: Option C

A 22 year old female is noted to have a reduced upper to lower body segment ratio, positive Walker and Steinberg signs, and pectus carinatum. Her father died suddenly at the age of 34. She is subsequently diagnosed with Marfan’s syndrome. Which of the following was the likely cause of death of her father?

  • A. Mitral valve prolapse
  • B. Aortic valve regurgitation
  • C. Myocardial infarction
  • D. Aortic dissection
  • Correct Answer: Option D

A 55 year old male complains of increasing dyspnea on exertion and orthopnea. His physical examination reveals an S3 heart sound, pulmonary rales, jugular venous distension, and lower extremity edema. Coronary angiography is normal. An echocardiogram confirms an ejection fraction of 5% indicating severe congestive heart failure and dilated cardiomyopathy. Which of the following could explain the above findings?

  • A. A history of heroin abuse
  • B. Heavy alcohol use
  • C. Tuberculosis infection
  • D. Prior chemotherapy with cisplatin
  • Correct Answer: Option B